Prove that the map $nmapsto 1/n, infty mapsto 0$ is a homeomorphism












1












$begingroup$


Consider the set $N=mathbb Ncup{infty}$ together with the following topology: a subset $U$ of $N$ is open if either $inftynotin U$ or $Nsetminus U$ is finite.



I'm trying to show that $A={frac{1}{n}:nge 1}cup{0}$ with topology induced from $mathbb R$ is homeomorphic to $N$.



Define the map $f:Nto A, nmapsto 1/n$ for $nin mathbb N={1,2,dots}$ and $infty mapsto 0$. It is bijective because its inverse is given by $g: Ato N, 0mapsto infty, 1/nmapsto n (nge 1)$. So it suffices to show that $f$ and $g$ are continuous.



Continuity of $f$. It suffices to show the preimage of any basic open set is open. Any basic open set is of the form $Acap U$ where $Usubset mathbb R$ is open. If $U$ does not contain $0$, then its preimage doesn't contain $infty$, so it's open. Now assume $U$ contains $0$. We need to show its preimage has finite complement. How to prove this? My argument shows that this is not true: $N-f^{-1}(Acap U)=f^{-1}(mathbb R-(Acap U))$, and $mathbb R-(Acap U)$ is infinite (since $Acap U$ is at most countable).



Continuity of $g$. Let $Usubset N$ be open. First suppose $infty notin U$. Then $0notin g^{-1}(U)$, but I'm stuck here. The second case is when $N-U$ is finite. This doesn't tell me anything about openness of the preimage either...










share|cite|improve this question









$endgroup$












  • $begingroup$
    Use that $frac1n to 0$...
    $endgroup$
    – Henno Brandsma
    Dec 5 '18 at 3:10
















1












$begingroup$


Consider the set $N=mathbb Ncup{infty}$ together with the following topology: a subset $U$ of $N$ is open if either $inftynotin U$ or $Nsetminus U$ is finite.



I'm trying to show that $A={frac{1}{n}:nge 1}cup{0}$ with topology induced from $mathbb R$ is homeomorphic to $N$.



Define the map $f:Nto A, nmapsto 1/n$ for $nin mathbb N={1,2,dots}$ and $infty mapsto 0$. It is bijective because its inverse is given by $g: Ato N, 0mapsto infty, 1/nmapsto n (nge 1)$. So it suffices to show that $f$ and $g$ are continuous.



Continuity of $f$. It suffices to show the preimage of any basic open set is open. Any basic open set is of the form $Acap U$ where $Usubset mathbb R$ is open. If $U$ does not contain $0$, then its preimage doesn't contain $infty$, so it's open. Now assume $U$ contains $0$. We need to show its preimage has finite complement. How to prove this? My argument shows that this is not true: $N-f^{-1}(Acap U)=f^{-1}(mathbb R-(Acap U))$, and $mathbb R-(Acap U)$ is infinite (since $Acap U$ is at most countable).



Continuity of $g$. Let $Usubset N$ be open. First suppose $infty notin U$. Then $0notin g^{-1}(U)$, but I'm stuck here. The second case is when $N-U$ is finite. This doesn't tell me anything about openness of the preimage either...










share|cite|improve this question









$endgroup$












  • $begingroup$
    Use that $frac1n to 0$...
    $endgroup$
    – Henno Brandsma
    Dec 5 '18 at 3:10














1












1








1





$begingroup$


Consider the set $N=mathbb Ncup{infty}$ together with the following topology: a subset $U$ of $N$ is open if either $inftynotin U$ or $Nsetminus U$ is finite.



I'm trying to show that $A={frac{1}{n}:nge 1}cup{0}$ with topology induced from $mathbb R$ is homeomorphic to $N$.



Define the map $f:Nto A, nmapsto 1/n$ for $nin mathbb N={1,2,dots}$ and $infty mapsto 0$. It is bijective because its inverse is given by $g: Ato N, 0mapsto infty, 1/nmapsto n (nge 1)$. So it suffices to show that $f$ and $g$ are continuous.



Continuity of $f$. It suffices to show the preimage of any basic open set is open. Any basic open set is of the form $Acap U$ where $Usubset mathbb R$ is open. If $U$ does not contain $0$, then its preimage doesn't contain $infty$, so it's open. Now assume $U$ contains $0$. We need to show its preimage has finite complement. How to prove this? My argument shows that this is not true: $N-f^{-1}(Acap U)=f^{-1}(mathbb R-(Acap U))$, and $mathbb R-(Acap U)$ is infinite (since $Acap U$ is at most countable).



Continuity of $g$. Let $Usubset N$ be open. First suppose $infty notin U$. Then $0notin g^{-1}(U)$, but I'm stuck here. The second case is when $N-U$ is finite. This doesn't tell me anything about openness of the preimage either...










share|cite|improve this question









$endgroup$




Consider the set $N=mathbb Ncup{infty}$ together with the following topology: a subset $U$ of $N$ is open if either $inftynotin U$ or $Nsetminus U$ is finite.



I'm trying to show that $A={frac{1}{n}:nge 1}cup{0}$ with topology induced from $mathbb R$ is homeomorphic to $N$.



Define the map $f:Nto A, nmapsto 1/n$ for $nin mathbb N={1,2,dots}$ and $infty mapsto 0$. It is bijective because its inverse is given by $g: Ato N, 0mapsto infty, 1/nmapsto n (nge 1)$. So it suffices to show that $f$ and $g$ are continuous.



Continuity of $f$. It suffices to show the preimage of any basic open set is open. Any basic open set is of the form $Acap U$ where $Usubset mathbb R$ is open. If $U$ does not contain $0$, then its preimage doesn't contain $infty$, so it's open. Now assume $U$ contains $0$. We need to show its preimage has finite complement. How to prove this? My argument shows that this is not true: $N-f^{-1}(Acap U)=f^{-1}(mathbb R-(Acap U))$, and $mathbb R-(Acap U)$ is infinite (since $Acap U$ is at most countable).



Continuity of $g$. Let $Usubset N$ be open. First suppose $infty notin U$. Then $0notin g^{-1}(U)$, but I'm stuck here. The second case is when $N-U$ is finite. This doesn't tell me anything about openness of the preimage either...







general-topology continuity






share|cite|improve this question













share|cite|improve this question











share|cite|improve this question




share|cite|improve this question










asked Dec 5 '18 at 2:33









user531587user531587

25413




25413












  • $begingroup$
    Use that $frac1n to 0$...
    $endgroup$
    – Henno Brandsma
    Dec 5 '18 at 3:10


















  • $begingroup$
    Use that $frac1n to 0$...
    $endgroup$
    – Henno Brandsma
    Dec 5 '18 at 3:10
















$begingroup$
Use that $frac1n to 0$...
$endgroup$
– Henno Brandsma
Dec 5 '18 at 3:10




$begingroup$
Use that $frac1n to 0$...
$endgroup$
– Henno Brandsma
Dec 5 '18 at 3:10










1 Answer
1






active

oldest

votes


















1












$begingroup$

First note that $N$ is compact: let $mathcal{U}$ be an open cover of $N$. Let $U_0in mathcal{U}$ be an open set that contains $infty$. This means that $Nsetminus U_0$ is finite (as it cannot be of the first type of open set that does not contain $infty$). For every of the finitely many $n in N setminus U_0$ we pick some $U(n)in mathcal{U}$ that contains $n$ and then $U_0, {U(n): n in Nsetminus U_0}$ is the required finite subcover of $mathcal{U}$.



So as the codomain is Hausdorff (even metric), it suffices to show continuity of $f$ to see it is a homeomorphism, as it will be a closed map by compactness.



Let $O$ be open in ${frac{1}{n}: n ge 1} cup {0}$.



If $0 in O$ then as in the reals $frac{1}{n} to 0$, we know that for some $M in mathbb{N}$ we have that $n ge M$ implies $frac{1}{n} in O$, by the definition of convergence. So $f^{-1}[O]$ contains ${n: n ge M} cup {infty}$ which ensures that $N setminus f^{-1}[O] subseteq {n : n < M}$ is finite and so $f^{-1}[O]$ is open.



If $ 0 notin O$ then $infty notin f^{-1}[O]$ by definition, and so $f^{-1}[O]$ is also open. So in either case, $f$ is continuous. Note that we could have replaced ${frac{1}{n}: n ge 1} cup {0}$ by any infinite convergent sequence in the reals and its limit in the place of $0$. The checking of continuity is the same.






share|cite|improve this answer











$endgroup$













  • $begingroup$
    Does any infinite sequence really works? Or do we need it to be monotonic as well?
    $endgroup$
    – user531587
    Dec 5 '18 at 3:44










  • $begingroup$
    @user531587 convergent is all we need.
    $endgroup$
    – Henno Brandsma
    Dec 5 '18 at 3:50











Your Answer





StackExchange.ifUsing("editor", function () {
return StackExchange.using("mathjaxEditing", function () {
StackExchange.MarkdownEditor.creationCallbacks.add(function (editor, postfix) {
StackExchange.mathjaxEditing.prepareWmdForMathJax(editor, postfix, [["$", "$"], ["\\(","\\)"]]);
});
});
}, "mathjax-editing");

StackExchange.ready(function() {
var channelOptions = {
tags: "".split(" "),
id: "69"
};
initTagRenderer("".split(" "), "".split(" "), channelOptions);

StackExchange.using("externalEditor", function() {
// Have to fire editor after snippets, if snippets enabled
if (StackExchange.settings.snippets.snippetsEnabled) {
StackExchange.using("snippets", function() {
createEditor();
});
}
else {
createEditor();
}
});

function createEditor() {
StackExchange.prepareEditor({
heartbeatType: 'answer',
autoActivateHeartbeat: false,
convertImagesToLinks: true,
noModals: true,
showLowRepImageUploadWarning: true,
reputationToPostImages: 10,
bindNavPrevention: true,
postfix: "",
imageUploader: {
brandingHtml: "Powered by u003ca class="icon-imgur-white" href="https://imgur.com/"u003eu003c/au003e",
contentPolicyHtml: "User contributions licensed under u003ca href="https://creativecommons.org/licenses/by-sa/3.0/"u003ecc by-sa 3.0 with attribution requiredu003c/au003e u003ca href="https://stackoverflow.com/legal/content-policy"u003e(content policy)u003c/au003e",
allowUrls: true
},
noCode: true, onDemand: true,
discardSelector: ".discard-answer"
,immediatelyShowMarkdownHelp:true
});


}
});














draft saved

draft discarded


















StackExchange.ready(
function () {
StackExchange.openid.initPostLogin('.new-post-login', 'https%3a%2f%2fmath.stackexchange.com%2fquestions%2f3026518%2fprove-that-the-map-n-mapsto-1-n-infty-mapsto-0-is-a-homeomorphism%23new-answer', 'question_page');
}
);

Post as a guest















Required, but never shown

























1 Answer
1






active

oldest

votes








1 Answer
1






active

oldest

votes









active

oldest

votes






active

oldest

votes









1












$begingroup$

First note that $N$ is compact: let $mathcal{U}$ be an open cover of $N$. Let $U_0in mathcal{U}$ be an open set that contains $infty$. This means that $Nsetminus U_0$ is finite (as it cannot be of the first type of open set that does not contain $infty$). For every of the finitely many $n in N setminus U_0$ we pick some $U(n)in mathcal{U}$ that contains $n$ and then $U_0, {U(n): n in Nsetminus U_0}$ is the required finite subcover of $mathcal{U}$.



So as the codomain is Hausdorff (even metric), it suffices to show continuity of $f$ to see it is a homeomorphism, as it will be a closed map by compactness.



Let $O$ be open in ${frac{1}{n}: n ge 1} cup {0}$.



If $0 in O$ then as in the reals $frac{1}{n} to 0$, we know that for some $M in mathbb{N}$ we have that $n ge M$ implies $frac{1}{n} in O$, by the definition of convergence. So $f^{-1}[O]$ contains ${n: n ge M} cup {infty}$ which ensures that $N setminus f^{-1}[O] subseteq {n : n < M}$ is finite and so $f^{-1}[O]$ is open.



If $ 0 notin O$ then $infty notin f^{-1}[O]$ by definition, and so $f^{-1}[O]$ is also open. So in either case, $f$ is continuous. Note that we could have replaced ${frac{1}{n}: n ge 1} cup {0}$ by any infinite convergent sequence in the reals and its limit in the place of $0$. The checking of continuity is the same.






share|cite|improve this answer











$endgroup$













  • $begingroup$
    Does any infinite sequence really works? Or do we need it to be monotonic as well?
    $endgroup$
    – user531587
    Dec 5 '18 at 3:44










  • $begingroup$
    @user531587 convergent is all we need.
    $endgroup$
    – Henno Brandsma
    Dec 5 '18 at 3:50
















1












$begingroup$

First note that $N$ is compact: let $mathcal{U}$ be an open cover of $N$. Let $U_0in mathcal{U}$ be an open set that contains $infty$. This means that $Nsetminus U_0$ is finite (as it cannot be of the first type of open set that does not contain $infty$). For every of the finitely many $n in N setminus U_0$ we pick some $U(n)in mathcal{U}$ that contains $n$ and then $U_0, {U(n): n in Nsetminus U_0}$ is the required finite subcover of $mathcal{U}$.



So as the codomain is Hausdorff (even metric), it suffices to show continuity of $f$ to see it is a homeomorphism, as it will be a closed map by compactness.



Let $O$ be open in ${frac{1}{n}: n ge 1} cup {0}$.



If $0 in O$ then as in the reals $frac{1}{n} to 0$, we know that for some $M in mathbb{N}$ we have that $n ge M$ implies $frac{1}{n} in O$, by the definition of convergence. So $f^{-1}[O]$ contains ${n: n ge M} cup {infty}$ which ensures that $N setminus f^{-1}[O] subseteq {n : n < M}$ is finite and so $f^{-1}[O]$ is open.



If $ 0 notin O$ then $infty notin f^{-1}[O]$ by definition, and so $f^{-1}[O]$ is also open. So in either case, $f$ is continuous. Note that we could have replaced ${frac{1}{n}: n ge 1} cup {0}$ by any infinite convergent sequence in the reals and its limit in the place of $0$. The checking of continuity is the same.






share|cite|improve this answer











$endgroup$













  • $begingroup$
    Does any infinite sequence really works? Or do we need it to be monotonic as well?
    $endgroup$
    – user531587
    Dec 5 '18 at 3:44










  • $begingroup$
    @user531587 convergent is all we need.
    $endgroup$
    – Henno Brandsma
    Dec 5 '18 at 3:50














1












1








1





$begingroup$

First note that $N$ is compact: let $mathcal{U}$ be an open cover of $N$. Let $U_0in mathcal{U}$ be an open set that contains $infty$. This means that $Nsetminus U_0$ is finite (as it cannot be of the first type of open set that does not contain $infty$). For every of the finitely many $n in N setminus U_0$ we pick some $U(n)in mathcal{U}$ that contains $n$ and then $U_0, {U(n): n in Nsetminus U_0}$ is the required finite subcover of $mathcal{U}$.



So as the codomain is Hausdorff (even metric), it suffices to show continuity of $f$ to see it is a homeomorphism, as it will be a closed map by compactness.



Let $O$ be open in ${frac{1}{n}: n ge 1} cup {0}$.



If $0 in O$ then as in the reals $frac{1}{n} to 0$, we know that for some $M in mathbb{N}$ we have that $n ge M$ implies $frac{1}{n} in O$, by the definition of convergence. So $f^{-1}[O]$ contains ${n: n ge M} cup {infty}$ which ensures that $N setminus f^{-1}[O] subseteq {n : n < M}$ is finite and so $f^{-1}[O]$ is open.



If $ 0 notin O$ then $infty notin f^{-1}[O]$ by definition, and so $f^{-1}[O]$ is also open. So in either case, $f$ is continuous. Note that we could have replaced ${frac{1}{n}: n ge 1} cup {0}$ by any infinite convergent sequence in the reals and its limit in the place of $0$. The checking of continuity is the same.






share|cite|improve this answer











$endgroup$



First note that $N$ is compact: let $mathcal{U}$ be an open cover of $N$. Let $U_0in mathcal{U}$ be an open set that contains $infty$. This means that $Nsetminus U_0$ is finite (as it cannot be of the first type of open set that does not contain $infty$). For every of the finitely many $n in N setminus U_0$ we pick some $U(n)in mathcal{U}$ that contains $n$ and then $U_0, {U(n): n in Nsetminus U_0}$ is the required finite subcover of $mathcal{U}$.



So as the codomain is Hausdorff (even metric), it suffices to show continuity of $f$ to see it is a homeomorphism, as it will be a closed map by compactness.



Let $O$ be open in ${frac{1}{n}: n ge 1} cup {0}$.



If $0 in O$ then as in the reals $frac{1}{n} to 0$, we know that for some $M in mathbb{N}$ we have that $n ge M$ implies $frac{1}{n} in O$, by the definition of convergence. So $f^{-1}[O]$ contains ${n: n ge M} cup {infty}$ which ensures that $N setminus f^{-1}[O] subseteq {n : n < M}$ is finite and so $f^{-1}[O]$ is open.



If $ 0 notin O$ then $infty notin f^{-1}[O]$ by definition, and so $f^{-1}[O]$ is also open. So in either case, $f$ is continuous. Note that we could have replaced ${frac{1}{n}: n ge 1} cup {0}$ by any infinite convergent sequence in the reals and its limit in the place of $0$. The checking of continuity is the same.







share|cite|improve this answer














share|cite|improve this answer



share|cite|improve this answer








edited Dec 5 '18 at 5:11

























answered Dec 5 '18 at 3:09









Henno BrandsmaHenno Brandsma

107k347114




107k347114












  • $begingroup$
    Does any infinite sequence really works? Or do we need it to be monotonic as well?
    $endgroup$
    – user531587
    Dec 5 '18 at 3:44










  • $begingroup$
    @user531587 convergent is all we need.
    $endgroup$
    – Henno Brandsma
    Dec 5 '18 at 3:50


















  • $begingroup$
    Does any infinite sequence really works? Or do we need it to be monotonic as well?
    $endgroup$
    – user531587
    Dec 5 '18 at 3:44










  • $begingroup$
    @user531587 convergent is all we need.
    $endgroup$
    – Henno Brandsma
    Dec 5 '18 at 3:50
















$begingroup$
Does any infinite sequence really works? Or do we need it to be monotonic as well?
$endgroup$
– user531587
Dec 5 '18 at 3:44




$begingroup$
Does any infinite sequence really works? Or do we need it to be monotonic as well?
$endgroup$
– user531587
Dec 5 '18 at 3:44












$begingroup$
@user531587 convergent is all we need.
$endgroup$
– Henno Brandsma
Dec 5 '18 at 3:50




$begingroup$
@user531587 convergent is all we need.
$endgroup$
– Henno Brandsma
Dec 5 '18 at 3:50


















draft saved

draft discarded




















































Thanks for contributing an answer to Mathematics Stack Exchange!


  • Please be sure to answer the question. Provide details and share your research!

But avoid



  • Asking for help, clarification, or responding to other answers.

  • Making statements based on opinion; back them up with references or personal experience.


Use MathJax to format equations. MathJax reference.


To learn more, see our tips on writing great answers.




draft saved


draft discarded














StackExchange.ready(
function () {
StackExchange.openid.initPostLogin('.new-post-login', 'https%3a%2f%2fmath.stackexchange.com%2fquestions%2f3026518%2fprove-that-the-map-n-mapsto-1-n-infty-mapsto-0-is-a-homeomorphism%23new-answer', 'question_page');
}
);

Post as a guest















Required, but never shown





















































Required, but never shown














Required, but never shown












Required, but never shown







Required, but never shown

































Required, but never shown














Required, but never shown












Required, but never shown







Required, but never shown







Popular posts from this blog

Le Mesnil-Réaume

Ida-Boy-Ed-Garten

web3.py web3.isConnected() returns false always